A lecture course consists of 595 students. The students are

This topic has expert replies
Master | Next Rank: 500 Posts
Posts: 187
Joined: Tue Sep 13, 2016 12:46 am
A lecture course consists of 595 students. The students are to be divided into discussion sections, each with an equal number of students. Which of the following cannot be the number of students in a discussion section.

a) 17
b) 35
c) 45
d) 85
e) 119

Please assist with above problem.

User avatar
Legendary Member
Posts: 2663
Joined: Wed Jan 14, 2015 8:25 am
Location: Boston, MA
Thanked: 1153 times
Followed by:128 members
GMAT Score:770

by DavidG@VeritasPrep » Wed Oct 19, 2016 5:29 am
alanforde800Maximus wrote:A lecture course consists of 595 students. The students are to be divided into discussion sections, each with an equal number of students. Which of the following cannot be the number of students in a discussion section.

a) 17
b) 35
c) 45
d) 85
e) 119

Please assist with above problem.
We're basically looking for which of those numbers is NOT a factor of 595. So let's break it down.

595 = 5*119; 119 is a factor, so E is out
119 = 7 * 17; 17 is a factor, so A is out

Now we've got the prime factorization of 595: 5 * 7 * 17
5*7 = 35; 35 is a factor, so B is out.
5*17 = 85; 85 is a factor, so D is out.

That leaves us with C
Veritas Prep | GMAT Instructor

Veritas Prep Reviews
Save $100 off any live Veritas Prep GMAT Course

User avatar
Legendary Member
Posts: 2663
Joined: Wed Jan 14, 2015 8:25 am
Location: Boston, MA
Thanked: 1153 times
Followed by:128 members
GMAT Score:770

by DavidG@VeritasPrep » Wed Oct 19, 2016 5:31 am
alanforde800Maximus wrote:A lecture course consists of 595 students. The students are to be divided into discussion sections, each with an equal number of students. Which of the following cannot be the number of students in a discussion section.

a) 17
b) 35
c) 45
d) 85
e) 119

Please assist with above problem.
A shortcut: 5 + 9 + 5 = 19. Because 19 is not a multiple of 3, 595 is not a multiple of 3. Any number that had 45 as a factor would HAVE to be a multiple of 3, so we know that 45 is not a factor of 595. Answer is C
Veritas Prep | GMAT Instructor

Veritas Prep Reviews
Save $100 off any live Veritas Prep GMAT Course

GMAT/MBA Expert

User avatar
Elite Legendary Member
Posts: 10392
Joined: Sun Jun 23, 2013 6:38 pm
Location: Palo Alto, CA
Thanked: 2867 times
Followed by:511 members
GMAT Score:800

by [email protected] » Wed Oct 19, 2016 10:11 am
Hi alanforde800Maximus,

Prime Factorization works nicely on this question (as David has shown). If you're comfortable doing math by hand though, you might be able to 'brute force' this question in 30 seconds. Based on the description, we're looking for a number that is NOT divisible into 595. Once you find it, you're done. So, starting at Answer A, how long would it take you to divide in those numbers before you found the answer?

GMAT assassins aren't born, they're made,
Rich
Contact Rich at [email protected]
Image